Infinite set of positive integers - choose infinitely many to be relative primes or not












4












$begingroup$


Given a set of infinitely many positive integers. Is it always possible to find a subset of this set which contains infinitely many numbers such that any two numbers in this subset are relative primes or any two numbers in this subset have a greatest common divisor greater than 1?



There is a beautiful solution for this problem, my teacher told me that it is hard but you don’t have to use anything.



So I am looking for solutions not using well known theorems... thanks!










share|cite|improve this question











$endgroup$








  • 4




    $begingroup$
    Hint. Consider two cases. (1) Some prime divides infinitely many members of the set. (2) No prime divides infinitely many members of the set. I don't think I have to say more about case (1). In case (2), construct your subset by choosing one number at a time, so that each number you choose is relatively prime to all of the numbers already chosen.
    $endgroup$
    – bof
    Dec 29 '18 at 9:01
















4












$begingroup$


Given a set of infinitely many positive integers. Is it always possible to find a subset of this set which contains infinitely many numbers such that any two numbers in this subset are relative primes or any two numbers in this subset have a greatest common divisor greater than 1?



There is a beautiful solution for this problem, my teacher told me that it is hard but you don’t have to use anything.



So I am looking for solutions not using well known theorems... thanks!










share|cite|improve this question











$endgroup$








  • 4




    $begingroup$
    Hint. Consider two cases. (1) Some prime divides infinitely many members of the set. (2) No prime divides infinitely many members of the set. I don't think I have to say more about case (1). In case (2), construct your subset by choosing one number at a time, so that each number you choose is relatively prime to all of the numbers already chosen.
    $endgroup$
    – bof
    Dec 29 '18 at 9:01














4












4








4





$begingroup$


Given a set of infinitely many positive integers. Is it always possible to find a subset of this set which contains infinitely many numbers such that any two numbers in this subset are relative primes or any two numbers in this subset have a greatest common divisor greater than 1?



There is a beautiful solution for this problem, my teacher told me that it is hard but you don’t have to use anything.



So I am looking for solutions not using well known theorems... thanks!










share|cite|improve this question











$endgroup$




Given a set of infinitely many positive integers. Is it always possible to find a subset of this set which contains infinitely many numbers such that any two numbers in this subset are relative primes or any two numbers in this subset have a greatest common divisor greater than 1?



There is a beautiful solution for this problem, my teacher told me that it is hard but you don’t have to use anything.



So I am looking for solutions not using well known theorems... thanks!







infinity greatest-common-divisor set-partition






share|cite|improve this question















share|cite|improve this question













share|cite|improve this question




share|cite|improve this question








edited Dec 29 '18 at 8:57









bof

52.6k559121




52.6k559121










asked Dec 29 '18 at 8:52









Leo GardnerLeo Gardner

499111




499111








  • 4




    $begingroup$
    Hint. Consider two cases. (1) Some prime divides infinitely many members of the set. (2) No prime divides infinitely many members of the set. I don't think I have to say more about case (1). In case (2), construct your subset by choosing one number at a time, so that each number you choose is relatively prime to all of the numbers already chosen.
    $endgroup$
    – bof
    Dec 29 '18 at 9:01














  • 4




    $begingroup$
    Hint. Consider two cases. (1) Some prime divides infinitely many members of the set. (2) No prime divides infinitely many members of the set. I don't think I have to say more about case (1). In case (2), construct your subset by choosing one number at a time, so that each number you choose is relatively prime to all of the numbers already chosen.
    $endgroup$
    – bof
    Dec 29 '18 at 9:01








4




4




$begingroup$
Hint. Consider two cases. (1) Some prime divides infinitely many members of the set. (2) No prime divides infinitely many members of the set. I don't think I have to say more about case (1). In case (2), construct your subset by choosing one number at a time, so that each number you choose is relatively prime to all of the numbers already chosen.
$endgroup$
– bof
Dec 29 '18 at 9:01




$begingroup$
Hint. Consider two cases. (1) Some prime divides infinitely many members of the set. (2) No prime divides infinitely many members of the set. I don't think I have to say more about case (1). In case (2), construct your subset by choosing one number at a time, so that each number you choose is relatively prime to all of the numbers already chosen.
$endgroup$
– bof
Dec 29 '18 at 9:01










1 Answer
1






active

oldest

votes


















0












$begingroup$

This is an application of the infinite Ramsey theorem, see
https://en.wikipedia.org/wiki/Ramsey%27s_theorem



Color a pair blue if they are coprime and red if they are not.






share|cite|improve this answer









$endgroup$













  • $begingroup$
    You used a well known theorem, that’s what I wanted to avoid. But it is a simple beautiful solution anyway, so thanks
    $endgroup$
    – Leo Gardner
    Dec 29 '18 at 10:07












Your Answer








StackExchange.ready(function() {
var channelOptions = {
tags: "".split(" "),
id: "69"
};
initTagRenderer("".split(" "), "".split(" "), channelOptions);

StackExchange.using("externalEditor", function() {
// Have to fire editor after snippets, if snippets enabled
if (StackExchange.settings.snippets.snippetsEnabled) {
StackExchange.using("snippets", function() {
createEditor();
});
}
else {
createEditor();
}
});

function createEditor() {
StackExchange.prepareEditor({
heartbeatType: 'answer',
autoActivateHeartbeat: false,
convertImagesToLinks: true,
noModals: true,
showLowRepImageUploadWarning: true,
reputationToPostImages: 10,
bindNavPrevention: true,
postfix: "",
imageUploader: {
brandingHtml: "Powered by u003ca class="icon-imgur-white" href="https://imgur.com/"u003eu003c/au003e",
contentPolicyHtml: "User contributions licensed under u003ca href="https://creativecommons.org/licenses/by-sa/3.0/"u003ecc by-sa 3.0 with attribution requiredu003c/au003e u003ca href="https://stackoverflow.com/legal/content-policy"u003e(content policy)u003c/au003e",
allowUrls: true
},
noCode: true, onDemand: true,
discardSelector: ".discard-answer"
,immediatelyShowMarkdownHelp:true
});


}
});














draft saved

draft discarded


















StackExchange.ready(
function () {
StackExchange.openid.initPostLogin('.new-post-login', 'https%3a%2f%2fmath.stackexchange.com%2fquestions%2f3055662%2finfinite-set-of-positive-integers-choose-infinitely-many-to-be-relative-primes%23new-answer', 'question_page');
}
);

Post as a guest















Required, but never shown

























1 Answer
1






active

oldest

votes








1 Answer
1






active

oldest

votes









active

oldest

votes






active

oldest

votes









0












$begingroup$

This is an application of the infinite Ramsey theorem, see
https://en.wikipedia.org/wiki/Ramsey%27s_theorem



Color a pair blue if they are coprime and red if they are not.






share|cite|improve this answer









$endgroup$













  • $begingroup$
    You used a well known theorem, that’s what I wanted to avoid. But it is a simple beautiful solution anyway, so thanks
    $endgroup$
    – Leo Gardner
    Dec 29 '18 at 10:07
















0












$begingroup$

This is an application of the infinite Ramsey theorem, see
https://en.wikipedia.org/wiki/Ramsey%27s_theorem



Color a pair blue if they are coprime and red if they are not.






share|cite|improve this answer









$endgroup$













  • $begingroup$
    You used a well known theorem, that’s what I wanted to avoid. But it is a simple beautiful solution anyway, so thanks
    $endgroup$
    – Leo Gardner
    Dec 29 '18 at 10:07














0












0








0





$begingroup$

This is an application of the infinite Ramsey theorem, see
https://en.wikipedia.org/wiki/Ramsey%27s_theorem



Color a pair blue if they are coprime and red if they are not.






share|cite|improve this answer









$endgroup$



This is an application of the infinite Ramsey theorem, see
https://en.wikipedia.org/wiki/Ramsey%27s_theorem



Color a pair blue if they are coprime and red if they are not.







share|cite|improve this answer












share|cite|improve this answer



share|cite|improve this answer










answered Dec 29 '18 at 9:19









A. PongráczA. Pongrácz

6,1171929




6,1171929












  • $begingroup$
    You used a well known theorem, that’s what I wanted to avoid. But it is a simple beautiful solution anyway, so thanks
    $endgroup$
    – Leo Gardner
    Dec 29 '18 at 10:07


















  • $begingroup$
    You used a well known theorem, that’s what I wanted to avoid. But it is a simple beautiful solution anyway, so thanks
    $endgroup$
    – Leo Gardner
    Dec 29 '18 at 10:07
















$begingroup$
You used a well known theorem, that’s what I wanted to avoid. But it is a simple beautiful solution anyway, so thanks
$endgroup$
– Leo Gardner
Dec 29 '18 at 10:07




$begingroup$
You used a well known theorem, that’s what I wanted to avoid. But it is a simple beautiful solution anyway, so thanks
$endgroup$
– Leo Gardner
Dec 29 '18 at 10:07


















draft saved

draft discarded




















































Thanks for contributing an answer to Mathematics Stack Exchange!


  • Please be sure to answer the question. Provide details and share your research!

But avoid



  • Asking for help, clarification, or responding to other answers.

  • Making statements based on opinion; back them up with references or personal experience.


Use MathJax to format equations. MathJax reference.


To learn more, see our tips on writing great answers.




draft saved


draft discarded














StackExchange.ready(
function () {
StackExchange.openid.initPostLogin('.new-post-login', 'https%3a%2f%2fmath.stackexchange.com%2fquestions%2f3055662%2finfinite-set-of-positive-integers-choose-infinitely-many-to-be-relative-primes%23new-answer', 'question_page');
}
);

Post as a guest















Required, but never shown





















































Required, but never shown














Required, but never shown












Required, but never shown







Required, but never shown

































Required, but never shown














Required, but never shown












Required, but never shown







Required, but never shown







Popular posts from this blog

Le Mesnil-Réaume

Ida-Boy-Ed-Garten

web3.py web3.isConnected() returns false always